La transformación pasiva de los campos de David Tong está mal

La definición de transformación activa de David Tong es clara. Bajo la transformación activa, las coordenadas (vectores base) no se modifican, sino el campo. Denoto los campos antiguo y nuevo como ϕ y ϕ . Entonces por transformación activa

ϕ ( X ) = ϕ ( λ 1 X ) .

Tenga en cuenta que he puesto el número primo en el campo y no la coordenada ya que el campo se cambia (rota) y no el sistema de coordenadas.

Para la transformación pasiva Tong escribe

ϕ ( X ) = ϕ ( λ X ) . Esto me parece mal. Cambio el sistema de coordenadas de X a X y el campo en el antiguo sistema de coordenadas es ϕ ( X ) y en el nuevo es ϕ ( X ) . Como es un campo escalar, tengo ϕ ( X ) = ϕ ( X ) . Ahora X = λ X .

entonces obtengo

ϕ ( X ) = ϕ ( λ X ) . Que claramente no es lo mismo que el de Tong. Ahora creo que la ecuación de Tong no es correcta porque no puedo encontrar ninguna falla en mi definición. Puede alguien por favor explicar esto.

Además, algunos podrían escribir la ley de transformación correcta para un campo vectorial tanto para rotación activa como pasiva (tanto para un campo covariante como contravariante) teniendo en cuenta las siguientes convenciones (que tomo de GR). Mientras lo escribe, especifique la matriz con los cambios de campo, así como la matriz con la que cambia la coordenada, como he escrito anteriormente para un campo escalar.

Cuando A es un vector contravariante se transforma como

A = λ A

Cuando A es un vector covariante se transforma como

A = λ 1 A .

Si entiendo correctamente, las transformaciones anteriores son pasivas.

Editar:

Sé que si el vector base se transforma como mi = λ 1 mi dónde mi son nuevos vectores base y mi son viejos vectores de base, entonces las coordenadas se transforman como X = λ X . Y en GR sabemos que los covectores se transforman como vector base (con λ 1 ) mientras que las contravariantes se transforman en coordenadas similares (con λ ). Y esta es una transformación pasiva (como en Caroll)

( Utilice esta convención en todo momento )

Así que en GR tenemos A m = λ m v A v -> (1)

y A m = ( λ 1 ) m v A v

Ahora tengo estas preguntas específicas:

  1. En las transformaciones anteriores, los componentes del vector (o covector), i. mi A v cambiar. Pero ninguno de los libros (Caroll) menciona el cambio en los argumentos. Por qué es así. ¿Por qué no cambian los argumentos también? Mientras que también ha escrito la transformación para coordenadas .

Entonces debería (1) ser realmente

A m ( X ) = λ m v A v ( X )

-> A m ( λ X ) = λ m v A v ( X )

y A m ( X ) = ( λ 1 ) m v A v ( X )

-> A m ( λ X ) = ( λ 1 ) m v A v ( X )

porque recuerda que en mi notación las coordenadas cambian como X = λ X (así que acabo de reemplazar eso. Pero no está obteniendo el mismo resultado que el mío. ¿Me equivoco en el cálculo o la comprensión? ¿Debería ser eq (1) como arriba (con cambio en las coordenadas representadas también o sin eso como libros GR denote) . Todo esto que he escrito es para transformación pasiva, porque las coordenadas han cambiado (y la forma funcional del campo/vector). Por favor, indique si lo que he escrito es correcto o no. Si es incorrecto, indique cuál la ecuación exacta o la comprensión es incorrecta.

  1. Ahora, en base a mi conocimiento de la transformación anterior de vectores base (con la matriz inversa), intento formar la transformación activa. Aquí cambian los campos/vectores y no las coordenadas. Así que realmente debería estar usando la matriz inversa λ 1 ( es correcto este razonamiento ) al escribir el cambio de las contravariantes (porque ahora sus componentes no han cambiado sino que han sido rotados. Y debería usar la matriz directa ( λ ) para las covariantes porque su transformación es inversa a la contravariante. Entonces

A m ( X ) = ( λ 1 ) m v A v ( X ) (No hay cambios en los argumentos ya que no se cambian).

y A m ( X ) = λ m v A v ( X ) (No hay cambios en los argumentos ya que no se cambian).

Nuevamente, indique si algo está mal aquí, precisamente la ecuación o suposición exacta.

  1. Por último sé por un campo escalar

Como es un campo escalar, tengo ϕ ( X ) = ϕ ( X ) . Ahora X = λ X .

entonces obtengo

ϕ ( X ) = ϕ ( λ X ) .

¿Qué es exactamente lo que está mal aquí? Entiendo su Transformación Activa, pero no puedo entender qué estoy haciendo mal con la Transformación Pasiva. Por favor, señale el error.

Tienes que ser un poco más cuidadoso acerca de cómo opera la transformación. Para la transformación pasiva, debe ser X = λ 1 X ; el resultado final de las transformaciones pasivas y activas tiene que ser el mismo ya que es solo una diferencia de punto de vista. En el caso activo, puede pensar en el campo siendo "ampliado" por λ . En el caso pasivo, piensas en "reducir" las coordenadas por λ 1 .
@Aaron no, ¿por qué debería ser? X = λ 1 X . Ver también aquí physics.stackexchange.com/questions/242756/…

Respuestas (1)

Considere la siguiente imagen.

ingrese la descripción de la imagen aquí

Tenemos un campo que es grande en el rectángulo rojo y pequeño en el resto. La función que nos dice el valor del campo en algún punto en las coordenadas X es ϕ ; eso es, ϕ ( X ) es el valor del campo en el punto etiquetado por coordenadas X = ( X 1 , X 2 ) .

Ahora realizamos una transformación activa correspondiente a una rotación del campo por 60 .

ingrese la descripción de la imagen aquí

Estamos usando las mismas coordenadas, pero después de la transformación ϕ ya no es la función que nos da los valores del campo. Debemos considerar una nueva función. ψ que está relacionado con el anterior a través de

ψ ( X ) = ϕ ( R 1 X )

dónde R es el 60 matriz de rotación.


Ahora consideramos una transformación pasiva en su lugar.

ingrese la descripción de la imagen aquí

Este es un cambio de coordenadas donde las nuevas coordenadas y = ( y 1 , y 2 ) están relacionados con los antiguos a través de

y = R X

Una vez que hemos adoptado la y -sistema coordinado, ϕ es una vez más la función incorrecta. Los valores de campo en un punto y están dadas por la función σ , que está relacionado con ϕ a través de

σ ( y ) = ϕ ( R y )

Usa las imágenes para convencerte de que si ϕ es grande en, digamos, X = ( 1 , 0 ) , entonces σ será grande en y = ( 1 2 , 3 2 ) . En otras palabras, σ ( 1 2 , 3 2 ) = ϕ ( 1 , 0 ) , lo cual es consistente con σ ( y ) = ϕ ( R y ) .


Para abordar su segunda pregunta, bajo una transformación activa (lineal), un campo vectorial V se transforma como V V dónde

V ( X ) = R V ( R 1 X )

En notación de componentes,

V m ( X ) = R     v m V v ( R 1 X )

Bajo una transformación pasiva (lineal) definida por un cambio de coordenadas y = R X ,

V ( y ) = R 1 V ( R y )
V m ( y ) = ( R 1 )     v m V v ( R y )

Ambas reglas de transformación se pueden ver de inmediato reemplazando los rectángulos en mis dibujos con flechas.


En respuesta a la edición, el error que está cometiendo es decir que ϕ ( X ) = ϕ ( X ) y luego reemplazando X = R X . Esto no es correcto. Por favor, lea el ejemplo que le di. Si las coordenadas se giran por 60 , entonces X = ( 1 , 0 ) tendrá nuevas nuevas coordenadas dadas por y = ( 1 2 , 3 2 ) . Por lo tanto, el nuevo campo evaluado en el punto ( 1 2 , 3 2 ) será igual al antiguo campo evaluado en el punto ( 1 , 0 ) , es decir

ϕ ( 1 2 , 3 2 ) = ϕ ( 1 , 0 )

Pero ( 1 2 , 3 2 ) = R ( 1 , 0 ) , no R 1 ( 1 , 0 ) . En general entonces, ϕ ( X ) = ϕ ( R X ) .

En las transformaciones anteriores, los componentes del vector (o covector), i. e Aν cambio. Pero ninguno de los libros (Caroll) menciona el cambio en los argumentos. Por qué es así. ¿Por qué no cambian los argumentos también? Mientras que también ha escrito la transformación para coordenadas.

Presumiblemente, Carroll no quería agregar demasiado a la notación. Pero si está transformando un campo vectorial , que toma diferentes valores en diferentes puntos del espacio-tiempo, entonces ciertamente debe tratar cada componente como una función y transformar los argumentos en consecuencia.

Ahora, en base a mi conocimiento de la transformación anterior de vectores base (con la matriz inversa), intento formar la transformación activa. Aquí cambian los campos/vectores y no las coordenadas. Así que realmente debería estar usando la matriz inversa λ−1 (¿es correcto este razonamiento?)

No, no es correcto. Si la configuración del campo se rota por 60 , entonces la dirección del vector también debe rotarse por 60 , como en mi diagrama.

Por último sé por un campo escalar. Como es un campo escalar, tengo ϕ′(x′)=ϕ(x). Ahora x′=λx.

De nuevo, esto está mal. Quizá sería más sencillo considerar una sola coordenada X y una coordenada escalada X = 2 X .

ingrese la descripción de la imagen aquí

cuando decimos X = 2 X , no queremos decir que la nueva etiqueta de un punto sea el doble de la etiqueta anterior. En cambio, queremos decir que el X las "marcas" están el doble de separadas que las X las marcas de verificación son.

Lo que esto significa es que el X coordenada de un punto es la mitad de la correspondiente X coordenada de ese punto. Como se puede ver en el diagrama, X = 3 corresponde a X = 1.5 , no X = 6 .

En última instancia, estás cometiendo el mismo error que si dijeras "1 metro es igual a 100 centímetros, por lo que la posición de un punto en metros es 100 × la posición del punto en centímetros". Lo tienes al revés.

Gracias por tu respuesta. Te entendí activo pero tengo dudas en los demás. Quería comentar aquí con las dudas pero creo que puedo explicar mejor las dudas en la pregunta misma. He agregado una edición bastante larga a la pregunta. Tenga paciencia conmigo y revise la edición, ya que le indicará en qué estoy exactamente confundido. Esta confusión ha persistido durante mucho tiempo. Sería realmente genial si pudiera resolverlo ahora (probablemente con una edición de su respuesta). Pero, por favor, eche un vistazo a la edición bastante larga . Debe transmitir exactamente todo lo que estoy confundido.
@Aaron Ver también aquí physics.stackexchange.com/questions/242756/… La respuesta hace lo que he hecho aquí transformación pasiva para campos escalares
Y son sus últimas ecuaciones correctas. x debería ser R inversa y no Ry.. ¿Son correctas las últimas 4 ecuaciones?
@Shashaank He editado mi respuesta para abordar sus preguntas adicionales. La respuesta que vinculó en su comentario anterior es incorrecta y está cometiendo el mismo error que usted. Mis últimas ecuaciones (antes de mi edición) son correctas.
@Shashaank, lo insto a que use los diagramas para dar sentido a estas transformaciones.
OK muchas gracias. Creo que entiendo tu punto ahora. lo he aceptado Supongo que está claro. Solo para confirmar -1) Su matriz R (solo R, no su inversa) es la matriz que cambia los vectores de base antiguos en vectores de base nuevos o es la matriz que da las coordenadas de un vector en una nueva base dadas las coordenadas del base antigua 2) En el 4º último eqn tienes R inversa entre paréntesis mientras que en el 3º último tienes R dentro de los paréntesis.
Tal vez podría simplemente agregar la transformación pasiva para un tensor de campo electromagnético con el cambio en los argumentos también. No he visto en ningún libro escribir cómo se chamgea el argumento. Agregue el tensor de campo EM para aclarar 2)
@Shashaank Es lo mismo que la regla de transformación de vectores, con una matriz de transformación adicional al frente. Para una transformación de coordenadas lineales X y = R X , tendrías F m v ( y ) = ( R 1 )     m α ( R 1 )     v β F α β ( R y ) .
@Shashaank Más generalmente, para una transformación de coordenadas no lineal y = F ( X ) , tendrías F m v ( y ) = X α y m X β y v F α β ( F ( y ) )